Đến nội dung

Hình ảnh

$\sqrt[m]{\frac{\sum a_{1}}{n}}\geq ...$


  • Please log in to reply
Chủ đề này có 4 trả lời

#1
nloan2k1

nloan2k1

    Thượng sĩ

  • Thành viên
  • 219 Bài viết

Với mọi số thực k<1; $a_{1},a_{2},...,a_{n} và m là các số thực dương$; n là số tự nhiên không âm

CMR: $\sqrt[m]{\frac{a_{1}+a_{2}+...+a_{n}}{n}}\geq \frac{\sqrt[m]{a_{1}}+\sqrt[m]{a_{2}}+...+\sqrt[m]{a_{n}}}{n}$

 



#2
NMDuc98

NMDuc98

    Sĩ quan

  • Thành viên
  • 314 Bài viết

Với mọi số thực k<1; $a_{1},a_{2},...,a_{n} và m là các số thực dương$; n là số tự nhiên không âm

CMR: $\sqrt[m]{\frac{a_{1}+a_{2}+...+a_{n}}{n}}\geq \frac{\sqrt[m]{a_{1}}+\sqrt[m]{a_{2}}+...+\sqrt[m]{a_{n}}}{n}$

Thế $k<1$ đóng vai trò gì bạn!


Nguyễn Minh Đức

Lặng Lẽ

THPT Lê Quảng Chí (Hà Tĩnh)


#3
Hoang Nhat Tuan

Hoang Nhat Tuan

    Hỏa Long

  • Thành viên
  • 974 Bài viết

Với mọi số thực k<1; $a_{1},a_{2},...,a_{n} và m là các số thực dương$; n là số tự nhiên không âm

CMR: $\sqrt[m]{\frac{a_{1}+a_{2}+...+a_{n}}{n}}\geq \frac{\sqrt[m]{a_{1}}+\sqrt[m]{a_{2}}+...+\sqrt[m]{a_{n}}}{n}$

Lại một bài sử dụng Holder:

BĐT tương đương:$\frac{a_1+a_2+...+a_n}{n}\geq \frac{(\sqrt[m]{a_1}+\sqrt[m]{a_2}+...+\sqrt[m]{a_n})^m}{n^m}$

 Đến đây dùng Holder thôi :D


Ngài có thể trói cơ thể tôi, buộc tay tôi, điều khiển hành động của tôi: ngài mạnh nhất, và xã hội cho ngài thêm quyền lực; nhưng với ý chí của tôi, thưa ngài, ngài không thể làm gì được.

#4
nloan2k1

nloan2k1

    Thượng sĩ

  • Thành viên
  • 219 Bài viết

Lại một bài sử dụng Holder:

BĐT tương đương:$\frac{a_1+a_2+...+a_n}{n}\geq \frac{(\sqrt[m]{a_1}+\sqrt[m]{a_2}+...+\sqrt[m]{a_n})^m}{n^m}$

 Đến đây dùng Holder thôi :D

Mình không muốn sử dụng Holder, vì đây chỉ là một bđt phụ thôi, sử dụng Holder chứng minh sẽ rất dài và mất thời gian.

Bài này có một cách sử dụng bđt AM-GM nhưng mình vẫn chưa nghĩ ra hướng làm!



#5
Truong Gia Bao

Truong Gia Bao

    Thiếu úy

  • Điều hành viên THPT
  • 511 Bài viết

Mình không muốn sử dụng Holder, vì đây chỉ là một bđt phụ thôi, sử dụng Holder chứng minh sẽ rất dài và mất thời gian.

Bài này có một cách sử dụng bđt AM-GM nhưng mình vẫn chưa nghĩ ra hướng làm!

Mình đang thắc mắc có điều kiện $ m \geq 1$ nữa không nhỉ? Bạn xem ở ĐÂY thử!


"Điều quan trọng không phải là vị trí ta đang đứng, mà là hướng ta đang đi."




0 người đang xem chủ đề

0 thành viên, 0 khách, 0 thành viên ẩn danh